Administración     

Olimpiadas de Matemáticas
Página de preparación y problemas

Selector
La base de datos contiene 1154 problemas y 775 soluciones.
OME Local
OME Nacional
OIM
OME Andalucía
Retos UJA
Problema 705
Hallar las cuatro últimas cifras de $3^{2004}$.
pistasolución 1info
Pista. Escribe $3^{2004}=(10-1)^{1002}$ y desarrolla por el binomio de Newton. Alternativamente, demuestra que $3^{500}\equiv 1\ (\text{mod}10000)$.
Solución. Tenemos que $3^{2004}=9^{1002}=(10-1)^{1002}$. Por lo tanto, podemos desarrollar por el binomio de Newton \[3^{2004}=1-\binom{1002}{1}\cdot 10+\binom{1002}{2}\cdot 10^2-\binom{1002}{3}\cdot 10^3+\ldots\] Todos los términos a partir de los puntos suspensivos van multiplicados por una potencia de $10$ de exponente mayor que $3$, luego no afecta a las cuatro últimas cifras. Ahora bien, tenemos que \begin{align*} \binom{1002}{1}&=1002,\quad \binom{1002}{2}=\frac{1002\cdot 1001}{2}=501\cdot 1001,\\ \binom{1002}{3}&=\frac{1002\cdot 1001\cdot 1000}{6}=167\cdot 1001\cdot 1000. \end{align*} Si hacemos estos productos fijándonos solo en las últimas cifras, tenemos que las dos últimas cifras de $\binom{1002}{2}$ son $01$ y la última de $\binom{1002}{3}$ es $0$, con lo que las cuatro últimas cifras de $3^{2004}$ son las mismas que las de $1-20+100-0=81$, es decir, la respuesta es $0081$.
Si crees que el enunciado contiene un error o imprecisión o bien crees que la información sobre la procedencia del problema es incorrecta, puedes notificarlo usando los siguientes botones:
Informar de error en enunciado Informar de procedencia del problema
Problema 673
Halla el producto de tres números primos $p,q,r$, sabiendo que: \[\left\{\begin{array}{l}r - q = 2p,\\ rq + p^2 = 676.\end{array}\right.\]
pistasolución 1info
Pista. Demuestra en primer lugar que $(p+q)^2=676$.
Solución. Sustituyendo $r=2p+q$ en la segunda ecuación nos queda $676=(2p+q)q+p^2=(p+q)^2$, luego tiene que ser $p+q=26$. Las formas de expresar $26$ como suma de dos primos son \[26=3+23=7+17=13+13=17+7=23+3\] y de todas ellas la única que cumple que $r=2p+q$ es primo es $p=3$ y $q=23$, con lo que $r=29$. El producto que nos piden es $pqr=3\cdot 23\cdot 29=2001$.
Si crees que el enunciado contiene un error o imprecisión o bien crees que la información sobre la procedencia del problema es incorrecta, puedes notificarlo usando los siguientes botones:
Informar de error en enunciado Informar de procedencia del problema
Problema 667
Un número natural es capicúa si al escribirlo en el sistema decimal se puede leer de igual forma de izquierda a derecha que de derecha a izquierda. Por ejemplo, $8$, $23432$ y $6446$ son capicúas. Sean $x_1\lt x_2 \lt\ldots\lt x_i\lt x_{i+1}\lt\ldots$ todos los números capicúas ordenados de menor a mayor. ¿Cuántos números primos distintos se pueden expresar como $x_{i+1}-x_i$ para algún $i$?
Sin pistas
Sin soluciones
info
Si crees que el enunciado contiene un error o imprecisión o bien crees que la información sobre la procedencia del problema es incorrecta, puedes notificarlo usando los siguientes botones:
Informar de error en enunciado Informar de procedencia del problema
Problema 663
Encontrar un número $N$ de cinco cifras diferentes y no nulas que sea igual a la suma de todos los números de tres cifras distintas que se pueden formar con las cinco cifras de $N$.
pistasolución 1info
Pista. Si llamamos $a,b,c,d,e$ a las cinco cifras de $N$, demuestra en primer lugar que la suma de los números de tres cifras que dice el enunciado es igual a $1332(a+b+c+d+e)$.
Solución. Llamemos $a,b,c,d,e$ a las cifras del número ordenadas de decenas de millar a unidades. En los números de tres cifras formados con $a,b,c,d,e$, cada una de ellas aparece $12$ veces en las centenas, $12$ en las decenas y $12$ en las unidades ya que una vez fijado un dígito en una posición hay $4$ posibles elecciones en otra posición y, para cada una de ellas, $3$ en la última posición. Esto nos dice que la suma de los números de tres cifras distintas que se pueden formar con $a,b,c,d,e$ es igual a \[1200(a+b+c+d+e)+120(a+b+c+d+e)+12(a+b+c+d+e)=1332(a+b+c+d+e),\] donde el primer término (con coeficiente $1200$) viene de la suma de las centenas, el segundo (con coeficiente $120$) viene de las decenas y el último (con coeficiente $12$) de las unidades. Entonces, la condición del enunciado se reescribe equivalentemente como \[1332(a+b+c+d+e)=N=10000a+1000b+100c+10d+e.\qquad (\star)\] Como $1332$ es múltiplo de $9$, deducimos que $N$ es múltiplo de $9$, luego la suma $a+b+c+d+e$ también lo es. Esto nos dice que $N$ es múltiplo de $9\cdot 1332=11988$. Los únicos múltiplos de $11988$ menores que $100000$ que tienen todos sus dígitos distintos son \[\{23976,35964,47952,71928,83916\}\] y de ellos el único que cumple $(\star)$ es $N=35964$, luego esta es la única solución.
Si crees que el enunciado contiene un error o imprecisión o bien crees que la información sobre la procedencia del problema es incorrecta, puedes notificarlo usando los siguientes botones:
Informar de error en enunciado Informar de procedencia del problema
Problema 656
Demostrar que hay una infinidad de pares $(x,y)$ de números naturales que satisfacen la ecuación \[2x^2-3x-3y^2-y+1=0.\]
pistasolución 1info
Pista. Reescribe la ecuación como $(2x-1)(x-1)=y(y-3)$, luego las soluciones racionales estarán en correspondencia con los números racionales $\frac{p}{q}$ tales que $2x-1=\frac{p}{q}y$ y $x-1=\frac{q}{p}(y-3)$. Resuelve este sistema lineal en las incógnitas $x$ e $y$ para deducir que es suficiente con escoger $p$ y $q$ enteros tales que $p^2-6q^2=1$. Este es un buen momento para darle un repaso a la ecuación de Pell.
Solución. La ecuación se puede plantear de forma equivalente como \[(2x-1)(x-1)=y(y-3).\] Por tanto, si hay una solución $(x,y)$, deberá haber un número racional $r=\frac{p}{q}$ tal que $2x-1=ry$ y $x-1=\frac{y-3}{r}$. Este es un sistema que se puede resolver de forma única en términos de $r$ y nos queda \[x=\frac{r^2-r-3}{r^2-6}=\frac{p^2-3pq-3q^2}{p^2-6q^2},\qquad y=\frac{r-2}{r^2-6}=\frac{pq-2q^2}{p^2-6q^2}.\] Para cada solución $(p,q)$ de la ecuación $p^2-6q^2=1$, lo anterior nos da una solución entera $(x,y)$ de la ecuación original. Podemos daros cuenta de que $p^2-6q^2=1$ es una ecuación de Pell cuyo parámetro $D=6$ no es un cuadrado perfecto, luego tiene infinitas soluciones. No obstante, hay que asegurar que $x$ e $y$ son positivos y que las soluciones de Pell dan soluciones distintas de la ecuación original.

Una forma de obtener soluciones de la ecuación de Pell es la siguiente (ver la nota). Partimos de una solución concreta no trivial, por ejemplo, tomaremos $p_1=5$, $q_1=2$ (esta solución se encuentra tras probar un poco). Se tiene entonces una sucesión de soluciones $(p_n,q_n)$ tomando $p_n$ y $q_n$ como los únicos números naturales que verifican $(5+2\sqrt{6})^n=p_n+q_n\sqrt{6}$. Podemos desarrollar \[p_n+q_n\sqrt{6}=(5+2\sqrt{6})(p_{n-1}+q_{n-1}\sqrt{6})=(5p_{n-1}+12q_{n-1})+(2p_{n-1}+5q_{n-1})\sqrt{6},\]

lo que nos da la recurrencia \[\left\{\begin{array}{l}p_n=5p_{n-1}+12q_{n-1},\\q_n=2p_{n-1}+5q_{n-1}.\end{array}\right.\] También tenemos así una recurrencia para $r_n=\frac{p_n}{q_n}$ haciendo lo siguiente: \[r_n=\frac{p_n}{q_n}=\frac{5p_{n-1}+12q_{n-1}}{2p_{n-1}+5q_{n-1}}=\frac{5\frac{p_{n-1}}{q_{n-1}}+12}{2\frac{p_{n-1}}{q_{n-1}}+5}=\frac{5r_{n-1}+12}{2r_{n-1}+5}.\] La función $f(x)=\frac{5x+12}{2x+5}$ cumple que $f(x)=x$ si y solo si $x=\pm\sqrt{6}$. Teniendo en cuenta que $\lim_{x\to\infty}f(x)=\frac{5}{2}\gt \sqrt{6}$, es fácil ver que \[\sqrt{6}\lt x\lt f(x)\lt\frac{5}{2}\quad \text{para todo } x\gt\sqrt{6}.\] De esta manera, teniendo en cuenta que $r_1=\frac{p_1}{q_1}=\frac{5}{2}$, se sigue que $\sqrt{6}\lt r_{n-1}\lt r_n\leq\frac{5}{2}$ para todo $n\in\mathbb{N}$. Esto tiene dos consecuencias que resuelven el problema.
  • En primer lugar, todos los $r_n$ son distintos, lo que nos dice que hay un número infinito de soluciones distintas al problema original. Esto es consecuencia de que $x$ e $y$ son funciones racionales de $r$ y una función racional no puede tomar el mismo valor para infinitos valores de la variable si no es idénticamente nula.
  • En segundo y último lugar, las expresiones $\frac{r^2-r-3}{r^2-6}$ y $\frac{r-2}{r^2-6}$ son positivas para todo $r\gt \sqrt{6}$ luego todas nuestras soluciones $r_n$ hacen a $x$ e $y$ positivos.

Nota. Lo que hemos usado es que la ecuación de Pell $p^2-6q^2=1$ puede verse como números $p+q\sqrt{6}$ de norma $1$ en el anillo $\mathbb{Z}[\sqrt{6}]$. La norma de $a+b\sqrt{6}\in\mathbb{Z}[\sqrt{6}]$, con $a,b\in\mathbb{Z}$, se define de hecho como \[N(a+b\sqrt{6})=a^2-6b^2=(a+b\sqrt{6})\cdot(a-b\sqrt{6}).\] En realidad, se define como el valor absoluto de lo anterior, pero esto no es relevante para lo que vamos a decir porque lo interesante es que $N$ es multiplicativa, es decir, \[N((a+b\sqrt{6})(c+d\sqrt{6}))=N(a+b\sqrt{6})N(c+d\sqrt{6}).\] Desarrollando esta ecuación, simplemente estamos diciendo que lo siguiente se cumple para cualesquiera $a,b,c,d\in\mathbb{Z}$: \[(ac+6bd)^2-6(ad+bc)^2=(a^2-6b^2)(c^2-6d^2).\] Por lo tanto, si encontramos un número $a+b\sqrt{6}$ de norma $1$, todas sus potencias de exponente natural también tendrán norma $1$. Esto es lo que se ha usado en el problema.

Si crees que el enunciado contiene un error o imprecisión o bien crees que la información sobre la procedencia del problema es incorrecta, puedes notificarlo usando los siguientes botones:
Informar de error en enunciado Informar de procedencia del problema
José Miguel Manzano © 2010-2024. Esta página ha sido creada mediante software libre